LSAT and Law School Admissions Forum

Get expert LSAT preparation and law school admissions advice from PowerScore Test Preparation.

 Administrator
PowerScore Staff
  • PowerScore Staff
  • Posts: 8950
  • Joined: Feb 02, 2011
|
#22715
Complete Question Explanation, compiled in part from instructor responses below.

Must Be True. The correct answer choice is (A)

This question ranks as one of the hardest LSAT questions ever, with only around 13% of all test takers selecting the correct answer choice.

This is all of the information we have about cod in the Grand Banks:

1. We estimate how much cod is available by averaging two estimates, which I'll call (X) and (Y).

2. (X) is based on the number of cod (apparently the total number) caught by research vessels at such-and-such time.

3. (Y) is based on the average number of tons of cod caught by commercial vessels per unit of "fishing effort."

4. (X) and (Y) have usually been pretty close to each other, but ...

5. ... lately, (Y) has been going up, and (X) has been going down at about the same rate.


Answer choice (A): this is the correct answer choice. Consider that the estimate is based on an average of two separate figures. Over the last ten years, one of the figures has been rising and the other has been falling, both at about the same rate. Essentially, the rise in one figure is offset by the drop in the other figure and so the estimate remains approximately the same. For example, say that in the first year both figures equaled 10 units each, and thus when averaged the estimate would be 10 units. Ten years later one figure has risen to 15 units while the other has dropped to 5 units. When averaged, these two figures also produce an average of 10 units.

Here's another example:

  • Traditionally both (X) and (Y) = 50.

    But for the last ten years, (X) has been dropping by 2 per year, while (Y) has been increasing by 2 per year. So our numbers have diverged, moving from 50/50 to:

    48/52, then
    46/54, then
    44/56, then
    42/58, then
    40/60, and so on.

    But the AVERAGE has remained the same, right? It's always 50, because (X) and (Y) mirror each other, moving by the same amount in opposite directions.


Answer choice (B) is incorrect since the commercial fishing figure is simply an average of the "number of tons of cod caught by various commercial vessels per unit of fishing effort." Since the commercial figure is an average, this gives us no indication of whether the number of commercial vessels has increased.

The problem with answer choice (B) is that we haven't the faintest idea about the size of the fishing fleet in the Grand Banks, and whether it's increased, decreased, or stayed the same over time. We know that the estimate of the cod stock based on commercial fishing has increased, but we don't even know anything about what's caused it to do so, much less about what that might tell us about fishing vessel count (all it means is that the amount of cod caught "per unit of fishing effort" has gone up, and that's it). So attempting to conclude anything about how many commercial boats in the Grand Banks are now fishing for cod compared to the past decade, especially saying it "increased substantially," is entirely impossible and (B) is gone as a result.


Answer Choice (C) is incorrect because there is no information given in the stimulus by which to draw that inference. The stimulus does not give any information on the accuracy of the two different methods. Since this is a must be true question we are looking for an inference that can be drawn from the information given in the stimulus.

Since the estimate is taken by averaging the two different measurements, and one has gone up over the last ten years by roughly the same amount that the other measurement went up, we know that the most recent estimate should be fairly close to the one that was taken ten years ago because the change in the two measurements will average each other out.


Answer choice (E) is incorrect. (E) is about 20 years ago—so there's simply no way to know any details about this unmentioned period in time and what the cod situation was like back then, and so we definitely do not know how much cod we had twenty years ago. All we know is that the two methods for estimating that amount used to be close to one another, and in the last decade they've been moving in different directions at about the same rate.

For example, let's say that 10 years ago, both the estimate from the research vessels and the estimate from the commercial fishing vessels totaled 10 units. The average of 10 and 10, then, is 10. Ten years later, the research vessel estimate is decreasing so let's say it's now at 5 units. But the fishing vessel estimate is increasing by about the same amount so let's say it's now at 15 units. The average of 5 and 15 is still 10.

Therefore, if both estimates used to be about the same and now one is increasing at about the same rate as the other is decreasing, the average of those two estimates is probably still about the same as it was 10 years ago. This leads us to answer choice (A).
 ellenb
  • Posts: 260
  • Joined: Oct 22, 2012
|
#6438
Dear Powerscore,

I want to make sure that I understand why ANSWER B is wrong. I looked at the explanations online:

"Answer choice (B) is incorrect since the commercial fishing figure is simply an average of the “number of tons of cod caught by various commercial vessels per unit of fishing effort.” Since the commercial figure is an average, this gives us no indication of whether the number of commercial vessels has increased."

However, it still does not make sense. I picked this answer and I want to make sure I understand for sure why it is incorrect. Is it a numbers to ratio type of question? It seems like it. However, I just want more details, I seem confused with the explanation.

Regards,

Ellen

PS: this question was very difficult, that's why I want to make sure I precisely understand why the answer that I picked being B is not correct. Since, starring at it and thinking over and over did not help I decided to ask Powerscore :). Hope I will get an answer promptly :) You guys rock at answering questions!! Yey Powerscore!! you rock!!!
 Justin Eleff
PowerScore Staff
  • PowerScore Staff
  • Posts: 19
  • Joined: Jul 27, 2012
|
#6456
As I read the question, this is all of the information we have about cod in the Grand Banks:

1. We estimate how much cod is available by averaging two estimates, which I'll call (X) and (Y).

2. (X) is based on the number of cod (apparently the total number) caught by research vessels at such-and-such time.

3. (Y) is based on the average number of tons of cod caught by commercial vessels per unit of "fishing effort."

4. (X) and (Y) have usually been pretty close to each other, but ...

5. ... lately, (Y) has been going up, and (X) has been going down at about the same rate.

Does all of this make sense so far? It might not because it's a lot of information, so if you need to, please take a moment to go back and reread the stimulus one more time.

Now, already we can see that answer choice (A) is going to be correct, because the final estimate (from number 1 above) is an average of two numbers, which we know (from number 5 above) have been increasing and decreasing, respectively, at the same rate.

Try converting this whole thing into numbers.

Traditionally both (X) and (Y) = 50.

But for the last ten years, (X) has been dropping by 2 per year, while (Y) has been increasing by 2 per year. So our numbers have diverged, moving from 50/50 to:

48/52, then
46/54, then
44/56, then
42/58, then
40/60, and so on.

But the AVERAGE has remained the same, right? It's always 50, because (X) and (Y) mirror each other, moving by the same amount in opposite directions.

Again, we now know answer choice (A) is correct. But why is answer choice (B) wrong?

Go back to where I listed out all of the information, and look at number 3. Estimate component (Y) is an average of tons of fish caught per unit of fishing effort. If one boat is out there fishing, we only want to know how it does PER UNIT. If two boats are out there, we only want to know how they do, on average, PER UNIT. If 20,000 boats are out there, we still only want to know how they do, on average, PER UNIT.

We do know that estimate component (Y) is increasing -- the stimulus says so -- but that just means that however many boats there are out there, on average they are catching more fish than they did before, per unit of effort. This has nothing to do with the total number of fish being caught (except inasmuch as that total is used to compute the average) and it has nothing to do with the total number of boats doing the fishing (except inasmuch as that total is used to compute the average, too). Indeed, you could change the number of commercial vessels to one or two or 20,000, or any other number, and it would not affect anything else about how the question worked. for example:
  • One boat? Average number of tons of cod caught per unit by the end of my list of diverging (X) and (Y) numbers, above: 60.

    Two boats? Average number of tons caught per unit: still 60.

    Twenty thousand boats? Average number of tons caught per unit: still 60!
So there's just no way to use the other information to conclude that we must have more (or fewer, or the same number of) boats.

Please let me know if that helps. Thanks!
 ellenb
  • Posts: 260
  • Joined: Oct 22, 2012
|
#8702
Dear Powerscore,

I read the explanation for this question, however, there are not explanations for the wrong answers (and why they are wrong) only A and B is explained. Could you go over the other wrong answers?

Thanks

Ellen
 Steve Stein
PowerScore Staff
  • PowerScore Staff
  • Posts: 1153
  • Joined: Apr 11, 2011
|
#8704
Hi Ellen,

Thanks for your question. In that one, one of the referenced estimates used has been increasing, while the other has been decreasing by about the same amount over the last decade. This supports answer choice A, which provides that this year's overall estimate average will probably be much like the one from ten years ago. The author provides no information about which of the two estimates is more accurate, which one should be used, or what the cod population was like 20 years ago.

I hope that's helpful--which of the other choices did you find appealing?

Let me know--thanks!

~Steve
 rameday
  • Posts: 94
  • Joined: May 07, 2014
|
#15031
For question 30 on page 1-85, why is E wrong and A correct? Is it because the stimulus only gives us information about the past 11 years so therefore E can't be correct because it goes back to twenty years ago and A is correct because its scope is limited to 11 years?


A
User avatar
 KelseyWoods
PowerScore Staff
  • PowerScore Staff
  • Posts: 1079
  • Joined: Jun 26, 2013
|
#15084
Hi A,

This is a very difficult question! You're on the right track with why answer choice (E) is incorrect. We definitely don't know how much cod we had twenty years ago. All we know is that the two methods for estimating that amount used to be close to one another, and in the last decade they've been moving in different directions at about the same rate.

For example, let's say that 10 years ago, both the estimate from the research vessels and the estimate from the commercial fishing vessels totaled 10 units. The average of 10 and 10, then, is 10. Ten years later, the research vessel estimate is decreasing so let's say it's now at 5 units. But the fishing vessel estimate is increasing by about the same amount so let's say it's now at 15 units. The average of 5 and 15 is still 10.

Therefore, if both estimates used to be about the same and now one is increasing at about the same rate as the other is decreasing, the average of those two estimates is probably still about the same as it was 10 years ago. This leads us to answer choice (A).

Also, I just wanted to make sure that you're aware of the detailed Logical Reasoning homework explanations available in the Online Student Center. If you look at those, you'll get explanations for why every correct answer is correct and every incorrect answer is incorrect for every LR question in your books. You're of course always welcome to ask us questions on this forum, especially if you're still confused after reading the explanations. But those explanations are a great resource for you as well so I just wanted to make sure you had found them :)

Hope this helps! Good luck as you're studying!

Best,
Kelsey
 adlindsey
  • Posts: 90
  • Joined: Oct 02, 2016
|
#37459
WTF!!! Another BS question! I also had B! And all I see is people justifying why A is write when there is nothing in the damn stimulus to prove it!!!!
 Jon Denning
PowerScore Staff
  • PowerScore Staff
  • Posts: 907
  • Joined: Apr 11, 2011
|
#37494
I see a number of people here getting lured into wrong answers (particularly B), so let me take a slightly different approach to start—an approach anyone reading this would also be wise to embrace!—and tackle this one from the perspective of what makes answers incorrect. For tough MBT, which this no doubt is (note that it's even worded more softly, as "most strongly supported"), that's often the distinguishing element between success and (frustrating) failure, as right answers are well-disguised...but wrong answers are always, ALWAYS justifiably eliminated.

The problem with answer choice (B) is that we haven't the faintest idea about the size of the fishing fleet in the Grand Banks, and whether it's increased, decreased, or stayed the same over time. We know that the estimate of the cod stock based on commercial fishing has increased, but we don't even know anything about what's caused it to do so, much less about what that might tell us about fishing vessel count (all it means is that the amount of cod caught "per unit of fishing effort" has gone up, and that's it). So attempting to conclude anything about how many commercial boats in the Grand Banks are now fishing for cod compared to the past decade, especially saying it "increased substantially," is entirely impossible and (B) is gone as a result.

Ditto (E), another popular trap: this introduces a time not touched on at all by the stimulus—the stimulus is about the past decade, while (E) is about 20 years ago—so there's simply no way to know any details about this unmentioned period in time and what the cod situation was like back then.

Why is (A) right? First, I'm never going to "just justify an answer because LSAC says it's right." I want to know why answers are credited as right and wrong, because come test day that's the only thing that matters: my ability to interpret this test in the same exact way that the test makers do.

Fortunately, (A), like every single right answer on this exam, is wholly and legitimately defensible. It is not just the best answer of the bunch, or borderline okay, it's the only one that even remotely works. And it works great (albeit cleverly)!

We know that the cod stock estimate is based on the average of two numbers, and the last sentence of the stimulus tells us that those two numbers have begun to separate. If the separation was random or if one was potentially outpacing the other in some fashion then sure, we'd be stuck. But it isn't random. It's not unknown. The last sentence also tells us that the two values being averaged have started to increase and decrease "by about the same amount," meaning as one goes up the other goes down proportionately. What does that mean for the average, if the two pieces used to find it are moving apart at equal rates? The average, the middle point, doesn't move. Start at a point and begin to move in opposite directions from it at the same speed. Where's your middle? Same spot that you started from.

And that's exactly what (A) says: the average (official estimate) probably hasn't changed much. The midpoint hasn't moved much if the pieces it's based on move away from it evenly. Simple math, and a valid answer.

So guys fight with this test all you want. But it'll get you absolutely nowhere. Seek to understand the test and why things are right and wrong and you'll find that you can then start to beat it. I promise.
 adlindsey
  • Posts: 90
  • Joined: Oct 02, 2016
|
#37537
I have seeked to understand this exam and it hasn't gotten me anywhere in the past three years

Get the most out of your LSAT Prep Plus subscription.

Analyze and track your performance with our Testing and Analytics Package.